Difference between revisions of "2013 AMC 12A Problems/Problem 25"
Armalite46 (talk | contribs) m (→Solution) |
|||
Line 7: | Line 7: | ||
==Solution== | ==Solution== | ||
− | Suppose <math>f(z)=z^2+iz+1=c=a+bi</math>. We look for <math>z</math> with <math>\ | + | Suppose <math>f(z)=z^2+iz+1=c=a+bi</math>. We look for <math>z</math> with <math>\operatorname{Im}(z)>0</math> such that <math>a,b</math> are integers where <math>|a|, |b|\leq 10</math>. |
First, use the quadratic formula: | First, use the quadratic formula: | ||
Line 15: | Line 15: | ||
Generally, consider the imaginary part of a radical of a complex number: <math>\sqrt{u}</math>, where <math>u = v+wi = r e^{i\theta}</math>. | Generally, consider the imaginary part of a radical of a complex number: <math>\sqrt{u}</math>, where <math>u = v+wi = r e^{i\theta}</math>. | ||
− | <math>Im (\sqrt{u}) = Im(\pm \sqrt{r} e^{i\theta/2}) = \pm \sqrt{r} \sin(\theta/2) = \pm \sqrt{r}\sqrt{\frac{1-\cos\theta}{2}} = \pm \sqrt{\frac{r-v}{2}}</math>. | + | <math>\operatorname{Im}(\sqrt{u}) = \operatorname{Im}(\pm \sqrt{r} e^{i\theta/2}) = \pm \sqrt{r} \sin(\theta/2) = \pm \sqrt{r}\sqrt{\frac{1-\cos\theta}{2}} = \pm \sqrt{\frac{r-v}{2}}</math>. |
Now let <math>u= -5/4 + c</math>, then <math>v = -5/4 + a</math>, <math>w=b</math>, <math>r=\sqrt{v^2 + w^2}</math>. | Now let <math>u= -5/4 + c</math>, then <math>v = -5/4 + a</math>, <math>w=b</math>, <math>r=\sqrt{v^2 + w^2}</math>. | ||
− | Note that <math>Im(z)>0</math> if and only if <math>\pm \sqrt{\frac{r-v}{2}}>\frac{1}{2}</math>. The latter is true only when we take the positive sign, and that <math>r-v > 1/2</math>, | + | Note that <math>\operatorname{Im}(z)>0</math> if and only if <math>\pm \sqrt{\frac{r-v}{2}}>\frac{1}{2}</math>. The latter is true only when we take the positive sign, and that <math>r-v > 1/2</math>, |
or <math>v^2 + w^2 > (1/2 + v)^2 = 1/4 + v + v^2</math>, <math>w^2 > 1/4 + v</math>, or <math>b^2 > a-1</math>. | or <math>v^2 + w^2 > (1/2 + v)^2 = 1/4 + v + v^2</math>, <math>w^2 > 1/4 + v</math>, or <math>b^2 > a-1</math>. |
Revision as of 17:26, 15 October 2013
Problem
Let be defined by . How many complex numbers are there such that and both the real and the imaginary parts of are integers with absolute value at most ?
Solution
Suppose . We look for with such that are integers where .
First, use the quadratic formula:
Generally, consider the imaginary part of a radical of a complex number: , where .
.
Now let , then , , .
Note that if and only if . The latter is true only when we take the positive sign, and that ,
or , , or .
In other words, for all , satisfies , with a unique solution . Therefore we need to count the number of ordered pairs such that integers , and that .
When , there is no restriction on so there are pairs;
when , there are pairs.
So there are in total.
See also
2013 AMC 12A (Problems • Answer Key • Resources) | |
Preceded by Problem 24 |
Followed by Last Question |
1 • 2 • 3 • 4 • 5 • 6 • 7 • 8 • 9 • 10 • 11 • 12 • 13 • 14 • 15 • 16 • 17 • 18 • 19 • 20 • 21 • 22 • 23 • 24 • 25 | |
All AMC 12 Problems and Solutions |
The problems on this page are copyrighted by the Mathematical Association of America's American Mathematics Competitions.